Difference between revisions of "1957 AHSME Problems/Problem 34"

(created solution page)
 
m (typo fix)
Line 12: Line 12:
  
 
== See Also ==
 
== See Also ==
{{AHSME 50p box|year=1957|ab=b|num-b=31|num-a=33}}
+
{{AHSME 50p box|year=1957|num-b=33|num-a=35}}
 
{{MAA Notice}}
 
{{MAA Notice}}
 
[[Category:AHSME]][[Category:AHSME Problems]]
 
[[Category:AHSME]][[Category:AHSME Problems]]

Revision as of 07:59, 26 July 2024

Problem

The points that satisfy the system $x + y = 1,\, x^2 + y^2 < 25$, constitute the following set:

$\textbf{(A)}\ \text{only two points} \qquad \\  \textbf{(B)}\ \text{an arc of a circle}\qquad \\  \textbf{(C)}\ \text{a straight line segment not including the end-points}\qquad\\  \textbf{(D)}\ \text{a straight line segment including the end-points}\qquad\\  \textbf{(E)}\ \text{a single point}$

Solution

$\fbox{\textbf{(C)} a straight line segment not including the end-points}$.

See Also

1957 AHSC (ProblemsAnswer KeyResources)
Preceded by
Problem 33
Followed by
Problem 35
1 2 3 4 5 6 7 8 9 10 11 12 13 14 15 16 17 18 19 20 21 22 23 24 25 26 27 28 29 30 31 32 33 34 35 36 37 38 39 40 41 42 43 44 45 46 47 48 49 50
All AHSME Problems and Solutions

The problems on this page are copyrighted by the Mathematical Association of America's American Mathematics Competitions. AMC logo.png